Nguyễn Thị Huyền Trang

chứng minh: \(\dfrac{n^5}{5}+\dfrac{n^3}{3}+\dfrac{7n}{15}\) \(\in Z\)với \(\forall n\in Z\)


Các câu hỏi tương tự
Vô danh
Xem chi tiết
coolkid
Xem chi tiết
Nguyễn thành Đạt
Xem chi tiết
2K9-(✎﹏ ΔΠGΣLS ΩҒ DΣΔTH...
Xem chi tiết
hoaan
Xem chi tiết
Toru
Xem chi tiết
Tuấn Hoàng
Xem chi tiết
Nguyễn Trần Lam Trúc
Xem chi tiết
Trần Minh Hiếu
Xem chi tiết